musah stands at the center of a rectangular field . He first takes 50 steps north, then 25 step west and finally 50 steps on a bearing of 315°. How far west and how far north is Musah final point from the center?

Answers

Answer 1

Answer:

85.36 far north from the center

10.36 far east from the center

Step-by-step explanation:

The extra direction taken in the north side is x

X/sin(360-315)=50/sin 90

Sin 90= 1

X/sin 45= 50

X= sin45 *50

X= 0.7071*50

X= 35.355 steps

X= 35.36

Then the west direction traveled

West =√(50² - 35.355²)

West = √(2500-1249.6225)

West= √1250.3775

West= 35.36 steps

But this was taken in an opposite west direction

From the center

He is 35.36 +50

= 85.36 far north from the center

And

25-35.36=-10.36

10.36 far east from the center


Related Questions

how many meters are in 250 centimeters

Answers

Answer:

2.5 meters

Step-by-step explanation:

Lisa built a rectangular flower garden that is 4 meters wide and has a perimeter of 26 meters.
What is the length of Lisa's flower garden?

Answers

Answer:

9 m

Step-by-step explanation:

Given that

Width of rectangular flower garden, w = 4 m

Perimeter of rectangular flower garden, p = 26 m

To find:

Length of Lisa's flower garden = ?

Solution:

First of all, let us understand perimeter, length and width of a rectangle.

Let ABCD be a rectangle. Please refer to the attached image.

Opposite sides of a rectangle are equal to each other.

AB = CD = Length  

Let the length be [tex]l[/tex] m.

BC = DA = Width = 4 m

Perimeter of a closed image is equal to the sum of all the sides of the image.

So, perimeter of ABCD:

[tex]p = AB + BC + CD + DA \\\Rightarrow \bold{ p = 2 \times (Length +Width)}[/tex]

[tex]26 = 2 \times (l +4)\\\Rightarrow 2l =26-8\\\Rightarrow \bold{l = 9 m}[/tex]

A triangle has vertices at (-4,-6),(3,3),(7,2). Rounded to two decimal places, which of the following is closest aporoximation of the perimeter of the triangle

Answers

Answer:

Perimeter= 29.12 unit

Step-by-step explanation:

Perimeter of the triangle is the length of the three sides if the triangle summef up together

Let's calculate the length of each side.

For (-4,-6),(3,3)

Length= √((3+4)²+(3+6)²)

Length= √((7)²+(9)²)

Length= √(49+81)

Length= √130

Length= 11.40

For (-4,-6),(7,2)

Length= √((7+4)²+(2+6)²)

Length= √((11)²+(8)²)

Length= √(121+64)

Length= √185

Length= 13.60

For (3,3),(7,2)

Length=√( (7-3)²+(2-3)²)

Length= √((4)²+(-1)²)

Length= √(16+1)

Length= √17

Length= 4.12

Perimeter= 4.12+13.60+11.40

Perimeter= 29.12 unit

PLEASE ANSWER ASAP!!!

Equation in the picture


Solve for r in the equation in the picture. You must use the LCD (Least Common Denominator) to simplify. You can also use cross products to solve.

Must show work

A. r = 19

B. r = 21

C. r = 25

D. r = 30



any unrelated answer will be reported​

Answers

Answer:

r = 19

Step-by-step explanation:

( r-5) /2 = ( r+2) /3

The least common denominator is 6

3/3 *( r-5) /2 = ( r+2) /3 * 2/2

3( r-5) /6 = 2( r+2) /6

Since the denominators are the same, the numerators are the same

3( r-5) = 2(r+2)

Distribute

3r -15 = 2r+4

Subtract 2r from each side

3r-2r -15 = 2r+4-2r

r-15 =4

Add 15 to each side

r-15+15 = 4+15

r = 19

if pentagon OPQRS is dilated by a scale factor or ?
from the origin to create O'P'Q'R'S: what is the ordered pair of point S'?

Answers

Answer:

Option (D) : (3.5, 8.75)

Given the graph, find an equation for the parabola.

Answers

Answer:

[tex]\Large \boxed{\sf \bf \ \ y=\dfrac{1}{16}(a-3)^2-2 \ \ }[/tex]

Step-by-step explanation:

Hello, please consider the following.

When the parabola equation is like

[tex]y=a(x-h)^2+k[/tex]

The vertex is the point (h,k) and the focus is the point (h, k+1/(4a))

As the vertex is (3,-2) we can say that h = 3 and k = -2.

We need to find a.

The focus  is (3,2) so we can say.

[tex]2=-2+\dfrac{1}{4a}\\\\\text{*** We add 2. ***}\\\\\dfrac{1}{4a}=2+2=4\\\\\text{*** We multiply by 4a. ***}\\\\16a=1\\\\\text{*** We divide by 16. ***}\\\\a=\dfrac{1}{16}[/tex]

So an equation for the parabola is.

[tex]\large \boxed{\sf y=\dfrac{1}{16}(a-3)^2-2 }[/tex]

Hope this helps.

Do not hesitate if you need further explanation.

Thank you

Calculate, correct to one decimal plice
the acute angle between the lines
3x - 4y + 5 = 0 and 2x + 3y -1 = 0
A. 70.69
B. 50.2
C. 39.8
D. 19.4​

Answers

Answer:

A. 70.69 is the correct answer.

Step-by-step explanation:

Given:

Two lines:

[tex]3x - 4y + 5 = 0 \\2x + 3y -1 = 0[/tex]

To find:

Angle between the two lines = ?

Solution:

Acute Angle between two lines can be found by using the below formula:

[tex]tan \theta = |\dfrac{(m_1 - m_2)}{ (1 + m_1m_2)}|[/tex]

Where [tex]\theta[/tex] is the acute angle between two lines.

[tex]m_1, m_2[/tex] are the slopes of two lines.

Slope of a line represented by [tex]ax+by+c=0[/tex] is given as:

[tex]m = -\dfrac{a}{b }[/tex]

So,

[tex]m_1 = -\dfrac{3}{- 4} = \dfrac{3}{4}[/tex]

[tex]m_2 = -\dfrac{2}{ 3}[/tex]

Putting the values in the formula:

[tex]tan \theta = |\dfrac{(\dfrac{3}{4}- (-\dfrac{2}{3}))}{ (1 + \dfrac{3}{4}\times (-\dfrac{2}{3 }))}|\\\Rightarrow tan \theta = |\dfrac{\dfrac{3}{4}+\dfrac{2}{3}}{ (1 -\dfrac{1}{2})}|\\\Rightarrow tan \theta = |\dfrac{\dfrac{17}{12}}{ \dfrac{1}{2}}|\\\Rightarrow tan \theta = \dfrac{17}{6}\\\Rightarrow \theta = tan^{-1}(\frac{17}{6})\\\Rightarrow \theta = \bold{70.69^\circ}[/tex]

So, correct answer is A. 70.69

Solve 2x+2y=6 and 3x-2y=11

Answers

Answer:

x = 17/5

y = -2/5

Step-by-step explanation:

2x + 2y = 6

3x - 2y = 11

sum both equations results

5x + 0 = 17

x = 17/5

2x + 2y = 6

2*17/5 + 2y = 6

34/5 + 2y = 6

2y = 6 - 34/5

2y = 30/5 - 34/5

2y = -4/5

y = (-4/5)/2

y = -2/5

verify:

3x - 2y = 11

3*17/5 - 2*-2/5 = 11

51/5 + 4/5 = 55/5

51 + 4 = 55

Does the function satisfy the hypotheses of the Mean Value Theorem on the given interval? f(x) = 4x2 − 3x + 2, [0, 2]

Answers

Answer:

Yes , it satisfies the hypothesis and  we can conclude that c = 1 is an element of [0,2]

c = 1 ∈ [0,2]

Step-by-step explanation:

Given that:

[tex]f(x) = 4x^2 -3x + 2, [0, 2][/tex]

which is read as the function of x = 4x² - 3x + 2 along the interval [0,2]

Differentiating the function with respect to x is;

f(x) = 8x - 3

Using the Mean value theorem to see if the function satisfies it, we have:

[tex]f'c = \dfrac{f(b)-f(a)}{b-a}[/tex]

[tex]8c -3 = \dfrac{f(2)-f(0)}{2-0}[/tex]

since the polynomial function is differentiated in [0,2]

[tex]8c -3 = \dfrac{(4(2)^2-3(2)+2)-(4(0)^2-3(0)+2)}{2-0}[/tex]

[tex]8c -3 = \dfrac{(4(4)-3(2)+2)-(4(0)-3(0)+2)}{2-0}[/tex]

[tex]8c -3 = \dfrac{(16-6+2)-(0-0+2)}{2-0}[/tex]

[tex]8c -3 = \dfrac{(12)-(2)}{2}[/tex]

[tex]8c -3 = \dfrac{10}{2}[/tex]

8c -3  = 5

8c = 5+3

8c = 8

c = 8/8

c = 1

Therefore, we can conclude that c = 1 is an element of [0,2]

c = 1 ∈ [0,2]

Find the equation of a parabola that has a vertex (3,5) and passes through the point (1,13).
Oy= -27 - 3)' +5
Oy=2(x + 3) - 5
Oy=2(0 - 3)' + 5
Oy= -3(2 – 3) + 5
PLEASE HELP ME!!

Answers

Answer:

y = 2(x - 3)² + 5

Step-by-step explanation:

The equation of a parabola in vertex form is

y = a(x - h)² + k

where (h, k) are the coordinates of the vertex and a is a multiplier

Here (h, k) = (3, 5), thus

y = a(x - 3)² + 5

To find a substitute (1, 13) into the equation

13 = a(1 - 3)² + 5 ( subtract 5 from both sides )

8 = 4a ( divide both sides by 4 )

a = 2, then

y = 2(x - 3)² + 5 ← equation of parabola in vertex form

What is the area of the house (including the drawing room, TV room, balcony, hallway, kitchen, and bedroom)?

Answers

Answer:

1256 i think

Step-by-step explanation:

Best Buy is currently selling the latest model of the iPad
Pro for $549.99. Since you are an employee there, you
receive a 5% discount. How much will the iPad Pro cost
you if you use your employee discount (before taxes).

Answers

Answer:

$522.49

Step-by-step explanation: 549.99*.05=27.50 (discount)

549.99-27.50=$522.49

Answer:

$522.49

Step-by-step explanation:

First, find the discount amount. You can do this by multiplying the original cost by the discount amount. A little trick for remembering to multiply instead of divide is to think "five percent of the original amount"

5% = 0.05

549.99 ⋅ 0.05 = 27.4995

That means the discount amount is $27.50

Subtract the discount amount from the original price

$549.99 - $27.50 = $522.49

The algebraic expression for the product of five and the cube of a number decreased by 40

Answers

Answer:

5a³ - 40

Step-by-step:

The algebraic expression is:

5a³ - 40

The algebraic expression for the product of five and the cube of a number decreased by 40 -

5 * x^3 - 40
* is multiplication sign.

Solve for W.
W/9 = g​

Answers

Answer:

W = 9 * g

Step-by-step explanation:

W/9 = g

W = 9 * g

The expression W/9 = g can be written as W = 9g after cross multiplication.

What is an expression?

It is defined as the combination of constants and variables with mathematical operators.

We have an expression:

W/9 = g

To solve for W

Make subject as W:

W = 9g

By cross multiplication.

Thus, the expression W/9 = g can be written as W = 9g after cross multiplication.

Learn more about the expression here:

brainly.com/question/14083225

#SPJ2

michaela has h hair ties. michaela's sister has triple the number of hair ties that michaela has. choose the expression that shows how many hair bows michaela's sister has

Answers

Answer:

[tex]S = 3 h[/tex]

Step-by-step explanation:

Let M represent Michaela hair tier and S represents Michaela  sister's

Given

M = h

S = Triple of M

Required

Determine an expression for S

From the given parameters, we have that;

S = Triple of M

Mathematically, this implies;

[tex]S = 3 * M[/tex]

Substitute h for M

[tex]S = 3 * h[/tex]

[tex]S = 3 h[/tex]

Hence, the expression for Michaela  sister' is [tex]S = 3 h[/tex]

Which expression is equivalent to 5y^3/(5y)^-2​

Answers

Answer:

5^3  y^5

125 y^5

Step-by-step explanation:

5y^3/(5y)^-2​

Distribute the exponent in the denominator

5y^3/(5 ^-2 y^-2)

A negative exponent in the denominator brings it to the numerator

5y^3 5 ^2 y^2​​

Combine like terms

5 * 5^2  * y^3 5^2

We know that a^b * a^c = a^(b+c)

5^(1+2) * y^( 3+2)

5^3  y^5

125 y^5

A charity organization is holding a food drive with a goal to collect at least 1,000 cans of
food by the end of the month. It currently has 565 cans from donations and is having an
event where 87 guests will attend and bring cans. Which solution set represents the
number of cans each guest must bring to meet the goal?
+
OA
++
0
1
2
3
4
5
6
7
8
9
10
---
+
OB. 4
+
0
1
2
3
4
5
6
7
8
9
10
OC.
+
1
2
3
5
6
7
8
9
10
OD. +
+
++
-
6
+
7.
+
0
1
2
3
4
5
8
9
10

Answers

Answer:

Each guest must bring 5 cans.

Step-by-step explanation:

1000-565=435

435/87=5

Need Help
Please Show Work​

Answers

Answer:

-36

Step-by-step explanation:

3*12=36

she is going down (negative) so, it is -36

not sure if this is what you are asking for, if not try this

0-12-12-12=-36

Find the domain and the range of the relation.
Find the domain of the relation. Select the correct choice below and fill in the answer box to
complete your choice.

O A. The domain is _
(Type your answer in interval notation.)
B. The domain is {_}
(Type an integer or a fraction. Use a comma to separate answers as needed.)
Find the range of the relation. Select the correct choice below and fill in the answer box to
complete your choice.
O A. The range is _
(Type an integer or a fraction. Use a comma to separate answers as needed.)
OB. The range is {_}

Answers

Answer:

1) the domain is all real numbers

2) the range is

[tex]y \geqslant 3[/tex]

Evaluate the expresión 6c-d when c=2 and d=10 I need help?

Answers

Answer:

the answer is 18

Step-by-step explanation:

8 is the answer

cindy was asked by her teacher to subtract 3 from a certain number and then divide the result by 6 instead, she subtracted 6 and then divided the result by 3 giving an answer of 25 what would her answer have been if she had worked the problem correctly?

Answers

The answer u been waiting for is -132

Answer:

13

Step-by-step explanation:

let the number be x

how Cindy worked it out :

(x -6) ÷ 3 = 25

x -6 = 75

x = 81

How she should have worked it out:

(x - 3) ÷ 6

(81 - 3) ÷ 6

78 ÷ 6 = 13

According to the United States Golf Association, the diameter of a golf ball should not be less than 42.67 millimeters. What is the estimate of this value rounded to the nearest tenth of a millimeter?

Answers

Answer:

42.7 mm

Step-by-step explanation:

To the nearest tenth of a mm, 42.67 mm would be 42.7 mm.

After estimate of this value rounded to the nearest tenth of a millimeter,

⇒ 42.67 ≈ 42.7

We have to given that,

According to the United States Golf Association, the diameter of a golf ball should not be less than 42.67 millimeters.

Hence, After estimate of this value rounded to the nearest tenth of a millimeter, we get;

⇒ 42.67

As, 7 is grater than 5, so we can add 1 to the tenth place.

⇒ 42.67 ≈ 42.7

Therefore, After estimate of this value rounded to the nearest tenth of a millimeter,

⇒ 42.67 ≈ 42.7

Learn more about the rounding number visit:

brainly.com/question/27207159

#SPJ2

Help me please please please please

Answers

Answer:

1.

d. (-14) + (-8)

2.

a. (-14) + 8

Step-by-step explanation:

(-14) - 8 is equal to (-14) + (-8) because we still add two negative values so the result wouldn't change.

(-14) - (-8) is equal to (-14) + 8 because there's two negative sign in front of 8 and two negative values multiplied makes a positive result.

Answer:

1. D  

2. A

Step-by-step explanation:

1.  It asks you what expression has the same value as (-14)-8.  All you need to do is find other equations that have the same value as that.  So the equation is -14-8.  IF a negative is outside a parenthesis with a positive number inside like -(+5), it is going to be -5.  If it's both negative: -(-5), it will be +5.  If it is both positive: +(+5), it is going to be +5.  

IMPORTANT!

- and + = -

- and - = +

+ and + = +

What we are looking for: -14-8

So choice A is (-14)+8 which is simplified to -14+8.  So, this one isn't right.

Choice B: 14-(-8)= 14+8.  So, it's incorrect.

Choice C: 14+(-8)= 14-8.  Again, it's not -14-8 so it's not right.

Choice D: (-14)+(-8)= -14-8.  This equation matches the one we are looking for!  So it's correct!

2. Same thing as number 1.  Let's simplify the equation it wants us to find first.  

(-14)-(-8)= -14+8

So -14+8 is what we are looking for.

Choice A: (-14)+8= -14+8.  It matches!  So it is correct.  Let's look at the other options anyway.

Choice B: 14-(-8)= 14+8.  Nope.  Not right.

Choice C: 14+(-8)= 14-8 because - always beats +.  So, this one is also incorrect.

Choice D: (-14)+(-8)= -14-8.  Oops, this is also wrong.  So choice A is the right answer.

Keep in mind, when you start getting questions like this with numbers inside the parenthesis as well, you want to remember the same rules for positive and negative, but also multiply the numbers together:

(When there is a number outside and inside a parentheses, multiply them.)

2(5)=10, CORRECT!    2+(5) is not 2 times 5.  It's whatever is closest to the parentheses, in this case being the positive sign.  So + and 5 is just 5!

IMPORTANT!

-2(-5)= - and - is positive, so positive (2 times 5).  Positive 10.

-2(+5)= - and + is negative, so negative (2 times 5). Negative 10.

+2(+5)= + and + is positive, so positive (2 times 5).  Positive 10.

Residents of four cities are able to vote in an upcoming regional election. A newspaper recently conducted a survey to gauge support for each of the two candidates. The results of the poll are shown in the two-way frequency table below.

Answers

Answer:

3 only

Step-by-step explanation:

Consider the statement, "The two cities with the highest number of respondents, both show more support for candidate A." In the total column, the two highest number of respondents are 471 and 463 which represent Carsonville and Appleton. For Carsonville, the number of respondents who prefer candidate A is 205, which is less than the number of respondents who prefer candidate B, 266. Therefore, this statement is not true.

Consider the statement, "The number of people who support candidate B in Carsonville is twice the number of people who support candidate B in New Thomas." In the table, the number of people who support candidate B in Carsonville is observed to be 266 and the number of people who support candidate B in New Thomas is 138. Since 266 is not equal to twice 138, this statement is not true.

Consider the statement, "More residents of Center City responded to the poll than the number who responded from New Thomas." In the total column, it can be observed that 350 people responded to the poll in Center City and 318 people responded to the poll in New Thomas. Since, 350 is greater than 318, this statement is true.

Consider the statement, "Overall, more residents support candidate A than candidate B." The bottom row of the table represents the total number of responses for each candidate. The number of people supporting candidate A is 797, which is less than the number of people supporting candidate B, 805. So, this statement is not true.

Therefore, the true statement is III only.

More residents of the center city responded to the pole than the number who responded from New Thomas, which is the only correct option. Option B. is correct.


Data given in the table shows the data of elections between two candidates among the different cities.

 

What is Statistic?

Statistics is the study of mathematics that deals with relations between comprehensive data.


I.The two cities with the highest number of respondents both show more support for candidate A. This statement is false because carsonville is the second highest support for A but it does not show more support for candidate A.

II.The number of people who support candidate B in Carsonville is twice the number of people who support candidate B in New Thomas. It is false

III. More residents of Center City responded to the pole than the number who responded from New Thomas. It is true.

IV. Overall, more residents support candidate A than candidate B. it is also false.

Thus, more residents of the center city responded to the pole than the number who responded from New Thomas, which is the only correct option. Option B. is correct.

Learn more about Statistics here:
https://brainly.com/question/23091366

#SPJ5

Billy has x marbles. Write an expression for the number of marbles the following have… a) Charlie has 5 more than Billy b) Danny has 8 fewer than Billy c) Eric has three times as many as Billy

Answers

Answer:

Charlie: 5 + xDanny: x - 8Eric: x × 3

please help me out! <3

Answers

Answer:

[tex]-1 \frac{3}{4}[/tex]

Step-by-step explanation:

Using this number line, we can plot our original number - [tex]\frac{3}{4}[/tex] (see picture attached)

Adding a negative is the same thing as subtracting - so we are subtracting [tex]2\frac{1}{2}[/tex] from  [tex]\frac{3}{4}[/tex].

To subtract this, we can break up [tex]2\frac{1}{2}[/tex]  into 3 parts: 1, 1, and [tex]\frac{1}{2}[/tex]. We can subtract each of these from the current number and see where we land up. (again see picture)

We land up at [tex]-1 \frac{3}{4}[/tex].

Hope this helped!

Write an equation showing the relationship between the lengths of the three sides of a right triangle.​

Answers

Answer:

Below

Step-by-step explanation:

First triangle)

This triangle is a right one so we will apply the pythagorian theorem.

● 25 is the hypotenus

● 25^2 = b^2 + 24^2

■■■■■■■■■■■■■■■■■■■■■■■■■■

Seconde triangle)

Again it's a right triangle

x is the hypotenus.

● x^2 = 12^2 +5^2

● 12^2 = x^2-5^2

■■■■■■■■■■■■■■■■■■■■■■■■■■

This is a right triangle

AC is the hypotenus.

● AC^2 = BC^2 + BA^2

Notice that: BC = BE+EC and BA=BD+DA

● AC^2 = (BE+EC)^2 + (BD+DA)^2

Answer:  2) b = 7       3) x = [tex]\sqrt{119}[/tex]

Step-by-step explanation:

Use Pythagorean Theorem: (leg₁)² + (leg₂)² = hypotenuse²

2) b² + 24² = 25²

   b² + 576 = 625

            b² = 49

         [tex]\sqrt{b^2}=\sqrt{49}[/tex]

            b = 7

3) 5² + x² = 12²

   25 + x² = 144

            x² = 119

        [tex]\sqrt{x^2}=\sqrt{119}[/tex]

           [tex]x=\sqrt{119}[/tex]

A line runs tangent to a circle at the point (4, 2). The line runs through the origin. Find the slope of the tangent line.

Answers

Answer:

Slope of the tangent line (m) = 1 / 2

Step-by-step explanation:

Given:

Point A = (4,2)

Origin point = (0,0)

Find:

Slope of the tangent line (m)

Computation:

Slope of the tangent line (m) = (y2-y1) / (x2-x1)

Slope of the tangent line (m) = (2-0) / (4-0)

Slope of the tangent line (m) = 2 / 4

Slope of the tangent line (m) = 1 / 2

Please Solve
F/Z=T for Z

Answers

Answer:

F /T = Z

Step-by-step explanation:

F/Z=T

Multiply each side by Z

F/Z *Z=T*Z

F = ZT

Divide each side by T

F /T = ZT/T

F /T = Z

Answer:

[tex]\boxed{\red{ z = \frac{f}{t} }}[/tex]

Step-by-step explanation:

[tex] \frac{f}{z} = t \\ \frac{f}{z} = \frac{t}{1} \\ zt = f \\ \frac{zt}{t} = \frac{f}{t} \\ z = \frac{f}{t} [/tex]

Point E lies within rectangle ABCD. If AE = 6, BE = 7, and CE = 8, what is the length of DE?

Answers

Answer:

[tex]\sqrt{51}[/tex] units.

Step-by-step explanation:

Point E is inside a rectangle ABCD.

Please refer to the attached image for the given statement and dimensions.

Given that:

Sides AE = 6 units

BE = 7 units and

CE = 8 units

To find:

DE = ?

Solution:

For a point E inside the rectangle the following property hold true:

[tex]AE^2+CE^2=BE^2+DE^2[/tex]

Putting the given values to find the value of DE:

[tex]6^2+8^2=7^2+DE^2\\\Rightarrow 26+64=49+DE^2\\\Rightarrow DE^2=100-49\\\Rightarrow DE^2=51\\\Rightarrow \bold{DE = \sqrt{51}\ units}[/tex]

Other Questions
12-(3-9) 3*3 help please why it is necessary to water the plant for experiment Can someone PLEASE help with this question? thank you What is the meaning of Delegation? a) Assigning the responsibility of completing a task to another. b) Being active. c) Being sincere. d) Being innovative. What is the first step in mathematical induction? Can someone explain to me what a derivative means? How do you find the derivative of f(x)=x^3+1? I need help pls now plleeeeeeeeaaassseeeee Marcus played 3 different activities this week after school. The time he spent playing is described below. Swam forof an hour Played soccer for 9/10 2/3 2/4 of an hour Jogged forof an hour Which statement correctly compares the times of 2 of his activities? Your investment club has only two stocks in its portfolio. $25,000 is invested in a stock with a beta of 0.8, and $40,000 is invested in a stock with a beta of 1.7. What is the portfolio's beta? Do not round intermediate calculations. Round your answer to two decimal places. explain the difference between legs and the hypotenuse of a right triangle Please help will give 5 stars with 1 thanks and 15 points This graph shows the US unemployment rate from August 2010 to November 2011.Sample Unemployment RateGraphUnemployment Rate10%80%6%Unemployment RateAug 10Jan 11Jun 11Nov 11This graph suggests unemployment in the United StatesO will continue to fall.O will continue to rise.O will remain the same.O will only change a little. f(x) = x2. What is g(x)? NEED HELP URGENT!! Which of the following was NOT a major cause of the new Red Scare in America? A. The Korean War B. The U-2 Plane incident C. The Alger Hiss case D. The Rosenberg spy trial find the value of x? please help Dyesss book was published when the war was still being fought. Beck and Burgos were interviewed in the 1990s, five decades after the war. How did Dyesss account of the Bataan Death March differ from Becks and Burgoss accounts? How were the accounts similar? Describe your answer in at least three sentences. In the lab, you shoot an electron towards the south. As it moves through a magnetic field, you observe the electron curving upward toward the roof of the lab. You deduce that the magnetic field must be pointing:_______. a. to the west. b. upward. c. to the north. d. to the east. e. downward. Pam Erickson Construction Company changed from the completed-contract to the percentage-of-completion method of accounting for long-term construction contracts during 2015. For tax purposes, the company employs the completed-contract method and will continue this approach in the future. (Hint: Adjust all tax consequences through the Deferred Tax Liability account.) The appropriate information related to this change is as follows.Pretax Income from:Percentage-of-Completion Completed-Contract Difference2014 $752,200 $586,700 $165,5002015 683,500 444,700 238,800(a) Assuming that the tax rate is 30%, what is the amount of net income that would be reported in 2015?Net income $(b) What entry(ies) are necessary to adjust the accounting records for the change in accounting principle? What was the ratio of per capita income in each of the following countries to that in the United States in the year 2010: a. Ethiopia b. Mexico c. India d. Japan Select the equation written in slope-intercept form that corresponds to the given slope and y-intercept. m=6 b=-2